Mathcenter Forum

Mathcenter Forum (https://www.mathcenter.net/forum/index.php)
-   ข้อสอบโอลิมปิก (https://www.mathcenter.net/forum/forumdisplay.php?f=28)
-   -   Warm Up ! (https://www.mathcenter.net/forum/showthread.php?t=1163)

passer-by 13 มิถุนายน 2005 05:55

Warm Up !
 
ใกล้เทศกาล สอบโอลิมปิกรอบแรกแล้ว สำหรับน้องๆ คนไหน ที่จะสอบแล้วยังรู้สึกไม่ฟิตพอ มาลอง Warm up ด้วยคำถาม ชิมลางจากหลายแหล่งที่มา ได้เลยครับ

1. หาจำนวนเต็มบวก m,n ทั้งหมดที่เป็นไปตามสมการ m2= 1!+2!+...+n!
2. มีจำนวนเต็ม n กี่จำนวน ในช่วง 1 ถึง 2005 ที่ \( \large (2\times 6 \times 10 \times...\times(4n-2)) \) หารลงตัวด้วย n!
3. สี่เหลี่ยม ABCD แนบในวงกลม โดยด้านที่สั้นสุด คือ AB ถ้า \( \huge \frac{Area \triangle BCD}{Area \triangle ABD}\) เป็นจำนวนเต็ม

และ AB,BC,CD,DA เป็นจำนวนเต็มต่างกันที่น้อยกว่าหรือเท่ากับ 10 หาค่ามากสุดที่เป็นไปได้ของ AB
4. หา b ทั้งหมดที่ทำให้ (x,y) เป็น real solutions ของระบบสมการ
\[\huge \sqrt{xy}=b^{b} \]
\[\huge log_{b}(x^{log_{b}y})+ log_{b}(y^{log_{b}x}) =4b^{4} \]
5. ให้ k เป็นจำนวนนับ พิสูจน์ว่า จะมี จำนวนเต็มบวก n ซึ่ง \(\huge k=\frac{1+\sqrt{8n-7}}{2} \)
6. ให้ P เป็นจุดบนด้าน BC ของสามเหลี่ยม ABC และ PC=2BP และมุม ABC=45 องศา และมุม APC= 60องศา หาขนาดมุม ACB
7. กำหนด f(x)=x4+ax3+bx2+cx+d โดย a,b,c,d เป็นจำนวนจริง ถ้า y=2x-1 ตัด f(x) ที่ x=1,2,3 หา f(0)+f(4)
8. หาค่า x2+y2+ z2 เมื่อ x,y,z เป็นจำนวนนับ และ \[\large 7x^{2}-3y^{2}+4z^{2}=8 \]
\[\large 16x^{2}-7y^{2}+9z^{2}=-3 \]

ระดับความยากของ 8 ข้อนี้ ก็ยังไม่ถึงขั้น hard core ซักเท่าไหร่ เอาไว้ให้น้องๆอุ่นเครื่องเฉยๆ นะครับ

สุดท้าย อยากขอความเห็น พี่ๆ เพื่อนๆ น้องๆ น่ะครับ คือ ผมกลัวว่า สุดท้ายก็จะมีแต่พี่ๆ เพื่อนๆ มือ pro มาถามเอง อธิบายเอง อย่างเคย
ถ้าเป็นไปได้ อยากให้บทบาทในส่วน ของพี่ๆมือโปรทั้งหลาย ที่เลยช่วงมัธยมมาแล้ว ทำหน้าที่เฉพาะตั้งคำถาม หรือหาคำถามมาเพิ่ม ส่วนหน้าที่ตอบคำถาม เอาไว้ให้น้องๆ มัธยม ซึ่งถ้าน้อง ติดขัดข้อไหน หรือจะ show วิธีทำข้อไหน พี่ๆอย่างเราๆค่อยมีหน้าที่ ตรวจสอบหรือแนะนำเพียงอย่างเดียว จะดีกว่ามั้ยครับ

nongtum 13 มิถุนายน 2005 06:39

ได้ครับ หากพี่ๆคิดไม่ออกหรือสงสัยข้อไหนก็ pm กันเองละกันนะครับ
เสริมให้อีกสองข้อ สำหรับคออสมการครับ
9. จงหาค่าสูงสุดของ \(\sqrt[\displaystyle{n}]{n}\) เมื่อ n เป็นจำนวนเต็มบวก
10. จงหาค่าของ
\[1+\frac{1}{\sqrt[4]{2}}+\frac{1}{\sqrt[4]{3}}+\ldots+\frac{1}{\sqrt[4]{10^{12}}} \]

Char Aznable 14 มิถุนายน 2005 20:51

ข้อ 1 ใช้ mod 5
ตอนนี้ข้อที่ทำได้แล้วก็มี 1,4,5,6,7,8,9 นะครับ
ที่เหลือจะพยายามทำ
ถ้าว่างจะมาโพสวิธีทำครับ

R-Tummykung de Lamar 14 มิถุนายน 2005 23:08

:eek: :eek: :eek:
สมเป็นบอส ทำได้เยอะมากๆๆ ... :D
ตัวข้าพเจ้าเองทำได้เพียงข้อเดียวเอง :mad:

passer-by 15 มิถุนายน 2005 06:20

เยี่ยมไปเลยครับ สำหรับน้อง Char Aznable แล้วจะรอดูวิธีทำนะครับ

สำหรับข้อ 3 เผื่อใคร ไม่ get โจทย์ ผมได้แก้ไข syntax บางส่วน แล้ว ลองกลับไปดูอีกทีนะครับ

แล้วก็ ต่อด้วย ข้อ 11-16 ครับ

11. ถ้า A แทนเซตของจำนวนเต็ม ในช่วง 1 ถึง 2004 ที่ทำให้อสมการ\[ \large \bigg \vert \frac{nx}{2004}-1 \bigg \vert < \frac{n}{2004} \] มีคำตอบเป็นจำนวนเต็ม แค่ 2 จำนวนต่างกัน
หา n(A)
12. หาค่า \[\large \sum_{n=1}^\infty \arctan \frac{2}{n^{2}} \]
13. หาคำตอบสมการ tan22x=2tan2xtan3x+1
14. หาจำนวนเต็มบวก a,b ที่สอดคล้องกับ a2- b2= b+1
15. ให้ an เป็นลำดับของจำนวนเต็ม โดย an-1+an=3n (n2) ถ้า a1=100 หา a1000
16. หากไม่ใช้ เครื่องคิดเลข เลข 5 ตัวนี้ ตัวใด มากสุด
\[\huge log_{2}3 \quad log_{3}5 \quad \sqrt{2} \quad \sqrt[3]{3} \quad 1.5\]

ไม่ยากเลยใช่มั้ยครับ ;)

nongtum 15 มิถุนายน 2005 08:06

ไม่ยากมากครับครั้งนี้ ไว้ครั้งหน้าหากกระแสตอบรับดีพอจะลองหาโจทย์การนับมาให้ทำกันครับ

17. ต่อด้าน AB และ AC ของสามเหลี่ยม ABC ไปทาง B และ C ตามลำดับ จากนั้นสร้างวงกลมสัมผัสด้าน BC และด้านที่ต่อไปทั้งสองด้าน ให้จุดศูนย์กลางของวงกลมนี้เป็น P หากมุม BAC มีขนาด 44 องศา จงหาขนาดของมุม BPC

18. จงแสดงว่า
a) \(100|7+7^2+\cdots+7^{2004}\)
b) \(186|5+5^2+\cdots+5^{150}\)

19. ให้จุด D อยู่บนด้าน AB ของสามเหลี่ยม ABC โดยที่จุด D อยู่ใกล้จุด A ให้จุด E อยู่บนส่วนของเส้นตรง DB และนอกจากนั้นให้ AD=DE=DC=CA, EC=EB จงหาขนาดของมุมภายในทั้งหมดของสามเหลี่ยม ABC

20. จงแก้(อ)สมการต่อไปนี้
a) \(log_{x}(2.5-\frac{1}{x})>1\) เมื่อ x เป็นจำนวนจริง
b) \(sin(\frac{\pi}{3}(x-\sqrt{x^2-3x+2}))=0\) เมื่อ x เป็นจำนวนเต็ม

21. จงหาจำนวนเต็มบวก n ที่น้อยที่สุดที่ทำให้ \(2^{2548}|n!\)

Edit1: แก้โจทย์ข้อ 19 ครับ

R-Tummykung de Lamar 15 มิถุนายน 2005 18:57

เย้ ได้เพิ่มอีกข้อแล้ว (แหะๆ) ขอลองทำ ข้อ 4 นะครับ พวกพี่ๆช่วยตรวจทานด้วยก็ดีนะครับ :D

R-Tummykung de Lamar 15 มิถุนายน 2005 19:18

ขอต่อด้วย ข้อที่ 15 นะครับ

nongtum 15 มิถุนายน 2005 23:13

ข้อ 15 ถูกแล้วครับ ส่วนข้อ 4 วิธีทำของน้องน่าสนใจดีครับ แต่ขอเสริมนิดหน่อย ตอนที่น้องใช้ AM-GM รวมทั้งก่อนหน้านั้น น่าหวาดเสียวเล็กน้อยว่าจะเจอการหารด้วยศูนย์(โชคดีที่ข้อนี้ไม่เจอ เพราะ b>0, b1) สำหรับวิธีที่ง่ายกว่าและไม่ปวดหัวกับการหารด้วยศูนย์เป็นดังนี้ครับ

แล้วจะรอคำตอบข้ออื่นครับ :D

passer-by 15 มิถุนายน 2005 23:54

คุณ nongtum อธิบายแทนผมไปเกือบหมดแล้ว สำหรับข้อ 4,15 Thanks you so much
จะมีเพิ่มเติมอีกนิด ก็คือ ข้อ 4 ของน้อง Tummykung
นอกจาก อาจเสี่ยงกับการหารด้วย 0 แล้ว ต้องระวัง A,B ติดลบด้วยนะครับ เพราะถึงแม้ x,y เป็นบวก แต่ log x, log y อาจเป็นลบ และทำให้ ใช้ AM-GM inequality ไม่ได้
ติเพื่อก่อ นะครับน้อง อย่าคิดมากนะ :)

รู้สึกว่ายังขาดคำถามเกี่ยวกับเมตริกซ์ งั้นผมแถมให้ 1 ข้อแล้วกัน

22. กำหนด
\[ \large A= \bmatrix{-4x+9 & -2x+6 & -2x-1 \\ 6x-11 & 2x-5 & 2x+5 \\ -2x+2 & -1 & x-2}\]หาผลบวกค่า x มากสุด 2 จำนวนแรก ที่ทำให้ เมตริกซ์ A เป็น singular matrix

ถ้า feedback ดี คงมีโจทย์ระลอกต่อไปให้ Warm up กันต่อ

Char Aznable 16 มิถุนายน 2005 19:26

ข้อ 11 จัดรูปสมการจะได้ 2004/n-1<x<2004/n+1 ถ้า2004หารด้วยnลงตัวจะมีคำตอบเพียงคำตอบเดียว ดังนั้นคำตอบคือจำนวนที่หาร2005ไม่ลงตัวทั้งหมด ซึ่งมีทั้งหมด
f(2004)= 1500 จำนวนครับ
ข้อ 13 จัดรูปสมการใหม่จะได้ (1+tan 2x tan3x)(1+tan x tan 2x )=0 ซึ่งเมื่อคิดแยกแต่ละอันแล้วไม่มีคำตอบ
ข้อ 14 ถ้า ab จะได้ b+1=a2-b20 เป็นไปไม่ได้
ดังนั้น a>b จะได้ b+1=a2-b2=(a-b)(a+b)>(a-b)(b+1)
จะได้ (a-b-1)(b+1)<0 ซึ่งเป็นไปไม่ได้ ดังนั้นสมการนี้ไม่มีคำตอบที่เป็นจำนวนเต็ม

Char Aznable 16 มิถุนายน 2005 19:50

ข้อ 2 พิจารณา จากโจทย์จัดรูปใหม่จะได้(2&times;6&times;10&times;...&times;(4n−2))/n! =2n!/n!n! = C(2n,n) ซึ่งเป็นจำนวนเต็มเสมอ นั่นคือ ทุกจำนวนนับ n ; n!หาร(2&times;6&times;10&times;...&times;(4n−2))ลงตัวเสมอครับ
ข้อ 3 จากสมบัติของพท.สามเหลี่ยมจะได้ AB*DAหารBC*CDลงตัว จากการไล่ค่าABที่เป็นไปได้ จะได้ABที่มากที่สุดคือ AB = 5ครับ
ข้อ 4 ผมทำแบบเดียวกับพี่ nongtum ครับ
ข้อ 5 เลือก n = k(k+1)/2+1
ข้อ 6 ผมใช้ตรีโกณมิติไล่หาด้านต่างๆในรูปของด้าน BP แล้วหาtan ACB ได้ (3+1)/22 จะได้มุมACB= 75
ข้อ 7 ให้g(x)=f(x)-(2x+1) จะได้ g(1)=g(2)=g(3)=0 แต่g(x)มีดีกรี4 ดังนั้น g(x) = (x-1)(x-2)(x-3)(x-k) จะได้ g(0)+g(4)=6k+(24-6k)=24 ดังนั้นf(0)+f(4)=g(0)+g(4)-1+7=30
ข้อ 8 กำจัดตัวแปรทิ้งแล้วไล่ค่าตัวแปรอีก2ตัวที่เหลือ จะได้คำตอบคือ (+-4,+-7,+-10) จะ
ได้ x2+y2+z2=165
ข้อ 9 ค่ามากสุดคือ 31/3 พิสูจน์โดยใช้ inductionครับ
ที่เหลือถ้ามีเวลาจะมาโพสต่อครับ
ปล.ผมสงสัยข้อ19อะครับ รบกวนพี่nongtumช่วยวาดรูปให้ดูหน่อยครับผมได้ECกับCBเป็นด้านเดียวกันครับ ไม่แน่ใจว่าใช่รึป่าวครับ

nongtum 16 มิถุนายน 2005 20:18

แก้โจทย์ข้อ 19 แล้วครับ (คงไม่ต้องวาดรูปให้ดูนา) รีบพิมพ์ไปนิด
ส่วนข้อเก้าน้องตอบถูกแล้วครับ แล้วจะมาขยายความทีหลัง ส่วนคำถามในส่วนที่คุณ Passer-by ตั้ง ขอเวลาเช็คแป๊บนึงครับ

R-Tummykung de Lamar 17 มิถุนายน 2005 00:29

ข้อ 11 ตรงนี้อะครับ
\[\frac{2004}{x}-1<x<\frac{2004}{x}+1 \]
ซึ่งคำตอบ คือจำนวนของจำนวนที่ไปหาร 2004 แล้วไม่ลงตัวใช่ไหมครับ ซึ่งผมคิดว่าไม่น่าจะใช่ \( \displaystyle{\phi(2004)} \) นะครับ (อีกอย่างผมได้ \( \displaystyle{\phi(2004)} \) = 664) เพราะว่า \( \displaystyle{\phi(2004)} \) คือจำนวนที่ ห.ร.ม. กับ 2004 แล้วได้ 1 ใช่ไหมครับ ในจำนวนนี้ไม่ได้รวมถึงจำนวนที่หารไม่ลงตัวแต่ ห.ร.มเป็นตัวอื่นด้วยครับ เช่น 8 รวมอยู่ด้วย \( \displaystyle{\phi(2004)} \) แต่ว่าไปหาร 2004 ไม่ลงตัว ทำให้ได้ x สองค่าเช่นกันครับ

\( \displaystyle{2004\ =\ 2^2\times3\times167}\)
ผมคิดว่าน่าจะเป็น 2004 - (2+1)(1+1)(1+1) = 1992 ครับ :D

ปล.ขอบคุณสำหรับการตรวจทานและข้อเสนอแนะของทุกๆคนครับ (ปกติก็ไม่คิดมากอยู่แล้ว อิอิ :D )
แล้วก็ขอทึ่งในความสามารถของคุณ Char Aznable อีกครั้งครับ :eek: :eek: :eek:

R-Tummykung de Lamar 17 มิถุนายน 2005 01:14

ข้อที่ 19 ใช้มุมในสามเหลี่ยม 2 ครั้ง ได้ \( \displaystyle{\widehat A\ =60^\circ\quad,\widehat B\ =15^\circ\quad,\widehat C\ =105^\circ} \)

ข้อที่ 22 ผมจัดรูปธรรมดา (เอาแถวสามคุณ 2 แล้วไปลบกับ แถวที่ 1 ..ไม่แน่ใจว่าเขียนว่า R1-2R3 รึเปล่า) แล้วหา det ตรงๆ ได้สมการคือ
\[ 4x^3-12x^2-19x+42=0\]
ได้ \( x\ =\ -2,\frac{3}{2},\frac{7}{2} \)
ผลบวกของสองจำนวนที่ มากที่สุดคือ \( \frac{3}{2}+\frac{7}{2}\ =\ 5\ \ \) ครับ :D

passer-by 17 มิถุนายน 2005 05:18

ข้อ 11,22 คำตอบของน้อง R-Tummykung De lamar ถูกแล้วครับ
และสำหรับ ข้อ 22 สิ่งที่น้อง กำลังทำ เกือบจะเป็น row operation แล้วครับ
syntax ของการทำ row operation ที่พี่ใช้ (ปัจจุบัน ไม่รู้ ยังเขียนประมาณนี้รึป่าว) จะเป็น
Rj cRi+Rj เมื่อ c0
ซึ่งหมายความว่า นำค่าคงที่ c ไปคูณแถวที่ i แล้วไปบวกแถวที่ j แล้ว replace แถวที่ j ด้วยค่านี้
ถ้าไม่อยากแก้สมการกำลังสาม ข้อนี้ อาจใช้ row operation 2 ครั้ง ตามลำดับดังนี้
1.1 R1 (1)(R2)+R1
1.2 R3 (1)(R1)+R3
และจะได้แถวสุดท้าย มี 0 ตั้ง 2 ตัว เวลา หา det สบายใจกว่าเดิมเยอะ

ส่วนคำตอบและคำอธิบายของน้อง Char Aznable โดยรวมก็ O.K. ครับ จะมีก็แต่

2.1 ข้อ 5 เกือบถูกแล้ว ต้องเลือก n= k(k-1)/2+1 นะครับ

2.2 ข้อ 6 ที่น้องเขียน tan ต้องเป็น sin ครับ แต่คำตอบถูกแล้ว ส่วนวิธีแบบที่ใช้เรขาคณิต อาจทำได้ดังนี้ครับ
จากโจทย์ กำหนด BP =x, PC=2x ลาก CD ตั้งฉากกับ AP ที่ D จะได้ DP=x ด้วย
ต่อไปก็ลาก BD ซึ่งจะทำให้ BD=AD=DC นั่นคือทำให้ สามเหลี่ยม ADC เป็นสามเหลี่ยมมุมฉากหน้าจั่วด้วย ดังนั้นมุม ACB= 30+45=75 องศา

2.3 ข้อ 14 สามารถอธิบายสั้นๆ แบบบรรทัดเดียวจบ ได้ดังนี้
เพราะ b2 < b2+b+1 =a2 <(b+1)2 ดังนั้น เป็นไปไม่ได้ที่จะมี a,b ดังกล่าว

เกือบลืมบอกไป 1 อย่าง เวลาสอบจริงๆ ถ้ามีพิสูจน์ อยากให้เขียนละเอียดระดับหนึ่ง และเรียบเรียงภาษาดีๆนะครับ เขียนใน board นี้ ย่อมากๆ ได้ แต่สอบจริง อย่าทำนะครับ
ที่ต้องมาเตือนเพราะว่า เคยคุยกับ คนที่สอน เด็ก สอวน. แล้วเขาบ่นๆ ว่า มีเด็กบางคน รู้แต่ อธิบายเป็นตัวหนังสือไม่รู้เรื่อง หรือเขียนสั้นเกินไป เดี๋ยวจะเสียคะแนนฟรีๆ นะคร้าบ

ตรวจงานเสร็จแล้ว ก็ต้อง ถามกันต่อไป

23. หาค่า x ทั้งหมดที่เป็นไปได้ของ \( \large x^{3}-3x=\sqrt{x+2} \)

24. พิสูจน์ว่า พหุนามที่มีรากทุกรากเป็นจำนวนจริง และ สัมประสิทธิ์ทุกตัว เป็น1 จะมีดีกรีอย่างมากเป็น 3

25. กำหนดลำดับ Fibonacci ที่มี F0=F1=1 หาจำนวนเต็ม n ทั้งหมด ในช่วง [0,100] ที่ทำให้ 13 หาร Fn ลงตัว

26. กำหนด \( \large sin10^{\circ}=0.1736 \) และ
\[ \large \sqrt{sin^{4}25^{\circ}+4cos^{2}25^{\circ}} - \sqrt{cos^{4}25^{\circ}+4sin^{2}25^{\circ}}=\sqrt{k} \] หา exact value ของ k จากข้อมูลที่มีอยู่

คราวนี้ ยากกว่าเดิมนิดหน่อย แต่คงไม่เกินความสามารถน้องๆใช่มั้ยครับ ;)

Char Aznable 17 มิถุนายน 2005 20:50

ขอบคุณพี่ๆมากครับ จะพยายามปรับปรุงเรื่องการเขียนและก็บางข้อรีบพิมเลยพิมผิดไปหน่อยครับ และก็ข้อ 11 ก็ขอบคุณtummykungที่บอกนะครับรีบไปหน่อย

nongtum 18 มิถุนายน 2005 05:48

ข้อ 19 ถูกแล้วครับ ถือว่าเป็น quickies แก้เซ็งตอนทำข้ออื่นไม่ได้ละกัน :p

ว่าแล้วก็เพิ่มโจทย์ต่อ ความยากง่ายคละกันเหมือนเดิม

27. เส้นตรงเส้นหนึ่งมีความชันเป็นลบและผ่านจุด C(1,1) และตัดแกน x และ y ที่จุด D และ E ตามลำดับ ให้จุด O เป็นจุดกำเนิด จงแสดงว่า พื้นที่DODE 2

28. เรียงเลขโดด 1,2,3,4,5,6 เป็นเลขหกหลัก จงหาความน่าจะเป็นที่เลขที่ได้ i) หาร 11 ลงตัว ii) หาร 9 แล้วได้เศษเป้นเลขคี่

29. มีจำนวนนับระหว่าง 1-102005 ทั้งหมดกี่ตัวที่ผลรวมของเลขโดดเป็น 2

30. จากรูป ให้สี่เหลี่ยมย่อย ABGH, BCFG, CDEF เป็นสี่เหลี่ยมจัตุรัส จงหาขนาดของมุม DIF

passer-by 20 มิถุนายน 2005 01:53

เพิ่ม Hint ให้ 1 ข้อแล้วกัน เผื่อจะได้มีไอเดีย
แต่จะไม่เชื่อ hint ก็ได้ครับ วิธีจะได้หลากหลาย

ส่วนข้อง่ายๆ ก็ยังเหลือ นะครับ เช่น ข้อ 17,30 ของคุณ nongtum :)

Char Aznable 20 มิถุนายน 2005 22:10

ข้อ 12 : จาก arctan 2/n2 = arctan (1-n) + arctan (1+n)
จะได้ ผลบวกในโจทย์คือ p/2

ปล.ขออภัยด้วยนะครับที่ผมไม่มีเวลาโพสวิธีทำแบบละเอียด

R-Tummykung de Lamar 21 มิถุนายน 2005 00:32

ข้อ 12 ผมได้ \( \displaystyle{\frac{3\pi}{4}}\) อะครับ
ผมใช้ความจริงที่คุณ Char Aznable บอกมาคือ \(\displaystyle{\arctan \frac{2}{n^2}\ =\ \arctan (n+1) + \arctan (1-n)} \) ผมดึง -1 ออกมา (เพราะ \(\arctan(-n)\ =\ -\arctan(n) \)
เป็น \(\displaystyle{\arctan \frac{2}{n^2}\ =\ \arctan (n+1) - \arctan (n-1)} \)
\( \displaystyle{\begin{array}{lcl} n\ =\ 1&\Rightarrow&\arctan (2)-\arctan(0)\\n\ =\ 2&\Rightarrow&\arctan (3)-\arctan(1)\\n\ =\ 3&\Rightarrow&\arctan (4)-\arctan(2) \\ \vdots \\n\ =\ k-1&\Rightarrow&\arctan (k)-\arctan(k-2) \\n\ =\ k&\Rightarrow&\arctan (k+1)-\arctan(k-1)\\n\ =\ k+1&\Rightarrow&\arctan (k+2)-\arctan(k)\end{array}} \)

ตัดๆกันหมด (ไม่แน่ใจว่าเรียกว่า Telescope รึเปล่า) เหลือ \( \displaystyle{-\arctan (0) - \arctan (1) + \arctan(k+1) + \arctan(k+2)}\)
แล้ว \( \displaystyle{\arctan (p) = \frac{\pi}{2}\quad เมื่อ\ p \rightarrow \infty \ \ (อันนี้คุณ M@gpie สอนครับ\ \ } \) :D )
ดังนั้น ตัวนี้มีค่าคือ \( \displaystyle{\ \ -0-\frac{\pi}{4}+\frac{\pi}{2}+\frac{\pi}{2}\ =\ \frac{3\pi}{4}\ \ ครับผม} \)


ขอกวาดข้อง่ายๆก่อนนะครับ (ยากๆทำไม่เป็น :D )
ข้อที่ 30
จากรูปได้ \( \displaystyle{D \widehat HE \ =\ \arctan (\frac{1}{3})} \) และ \(\ \ \displaystyle{A \widehat FH \ =\ \arctan (\frac{1}{2})} \)
ดังนั้น \(\ \ \displaystyle{D \widehat IF \ =\ A \widehat FH \ +D \widehat HE \ =\ \arctan (\frac{1}{2}) +\arctan (\frac{1}{3})} \)
ใช้ความจริง \( \displaystyle{\arctan (x)+\arctan (y)\ =\ \arctan \big(\frac{x+y}{1-xy}\big),xy<1} \)
ก็จะได้ \(\displaystyle{\arctan (\frac{1}{2}) +\arctan (\frac{1}{3}) \ =\ \arctan (1) \ =\ \frac{\pi}{4} } \)

nongtum 21 มิถุนายน 2005 02:36

ข้อ 30 ตอบถูกแล้วครับ จะขอเสริมวิธีไม้ตาย(เมื่อใช้วิธีอื่นแล้วคิดไม่ออก)โดยใช้ข้อนี้เป็นตัวอย่างดังนี้
หากมองว่าจุด H เป็นจุดกำเนิดและด้านของสี่เหลี่ยมจัตุรัสยาวหนึ่งหน่วย จะได้สมการของเส้นตรงทั้งสองเป็น \(l_1:y=\frac{1}{3}x\) และ \(l_2:y=-\frac{1}{2}x+1\) ซึ่งตัดกันที่ I(6/5,2/5) และสมการของเส้นตรงที่ตั้งฉากกับ \(l_2\) และผ่านจุดกำเนิดคือ y=2x ซึ่งตัดกับเส้นตรง \(l_2\) ที่จุด (2/5,4/5) ดังนั้น \(tan(A\hat IH)=\frac{\sqrt{16+4}/5}{\sqrt{16+4}/5}=1\) หรือมุม DIF มีขนาดเป็น 45 องศานั่นเอง

ขอสรุปข้อที่ยังไม่มีคนตอบ แยกตามคนออกโจทย์ ดังนี้
nongtum: 10,17,18,20,21,27,28,29,31 (ทุกข้อยกเว้นข้อ 10 ง่ายกว่าที่เห็นครับ ส่วนข้อสิบไม่จำเป็นต้องตอบแบบ exact ครับ)
passer-by: 16,23,24,25,26

ไว้ว่างๆจะมาเพิ่มโจทย์ต่อครับ (ข้อ 12 น่าจะตอบ \(-\frac{\pi}{4}\) มากกว่านา ยังไงรบกวนคุณ Passer-by มาตรวจการบ้านด้วยครับ)

passer-by 21 มิถุนายน 2005 03:45

คำอธิบายและคำตอบของน้อง Tummykung ถูกแล้วครับ
จริงๆ ข้อนี้ มีวิธี transform ได้หลายแบบ เช่น อาจจะให้
arctan(2/n2)=arctan( 1/(n-1)) - arctan(1/(n+1)) เมื่อ n2 ก็ได้

ส่วนที่คุณ nongtum สงสัยว่าทำไมไม่ใช่ -p/4 ผมว่า คำตอบนี้ ไม่ valid นะครับ เพราะ 2/n2 >0 ดังนั้น arctan( 2/n2) อยู่ในช่วง (0,p/2) ซึ่ง เมื่อนำมาบวกกัน infinite terms ยังไงก็ต้องไม่ติดลบครับ

ผมสังเกตได้อย่างหนึ่งว่า ถ้ามีคน 2 คน ใช้วิธีแบบน้อง Tummykung แต่จบด้วยรูปแบบต่างกัน คนแรกบวกกันแล้วตัดกันไปเรื่อยๆ ก็จะตอบ -p/4 (สงสัยคุณ nongtum น่าจะใช้วิธีนี้) กับอีกคนที่ take limit ให้ partial sum อย่างที่น้อง tummykung ทำตอนท้าย ก็จะตอบ 3p/4

แสดงว่า serie ตระกูล arctan ต้องรอบคอบพอสมควร เวลาสอบจริงๆ

สำหรับน้อง Char Aznable ที่ไม่ได้ post วิธีทำ อันนี้ไม่เป็นไรครับ เพราะยังไงการบ้านและงานมหาศาลจากโรงเรียนต้องมาอันดับหนึ่งครับ

อ้อ! เกือบลืมไป
1.1 ข้อ 1 ถ้าไม่อยากวุ่นวายกับ modulo ให้สังเกตว่า ตั้งแต่ n=4,5,6,... summation ทางขวาลงท้ายด้วย 3 เสมอ ดังนั้นไม่มี square ของ m ที่เป็นไปได้ ก็เลยมีทางเลือกแค่ n=1,2,3 ซึ่งเมื่อ ตรวจสอบทีละตัว ก็จะได้คำตอบครับ
1.2 ใครรู้วันสอบโอลิมปิก สสวท. รอบแรกของเลข บอกกันด้วยเน้อ เดี๋ยวจะ warm up กันเมามันเกินเหตุ

R-Tummykung de Lamar 21 มิถุนายน 2005 20:34

วันที่สอบสสวท.รอบแรกคือวันที่ 2-3 กรกฎาคม ครับผม ซึ่งผมไม่แน่ใจว่าเลขนี่ วันแรก หรือวันที่สองครับ :D

passer-by 26 มิถุนายน 2005 03:03

วันพฤหัสที่ 30 มิ.ย. จะมา post เฉลยข้อที่คั่งค้างของผมให้นะครับ เผื่อไว้ดูก่อนสอบจริงวันที่ 2-3 ก.ค. ดังนั้นตอนนี้ น้องๆอยากจะตอบข้อไหน ก็เชิญโดยด่วนนะครับ ยังรอตรวจคำตอบและให้คำแนะนำเหมือนเคยคร้าบผม ;)

R-Tummykung de Lamar 26 มิถุนายน 2005 12:32

งั้นผมขอทำ ข้อที่ 20 ก่อนนะครับ
a.) \( \displaystyle{จาก\ \ \log _x (2.5- \frac{1}{x})>1} \)
\( \displaystyle{\begin{array}{rcrcl} กรณีที่ 1
&0<x<1\ \ จะได้&2.5-\frac{1}{x}&<&x\\&&2x^2-5x+2&>&0&(คูณตลอดด้วย\ 2x\ ซึ่งมากกว่า\ 0)\\&&(2x-1)(x-2)&>&0\\&&x&\in& \big(0,\frac{1}{2}\big)\\กรณีที่ 2
&x>1&...\\&&(2x-1)(x-2)&<&0\\&&x&\in& \big(1,2\big) \end{array}} \)

\( \displaystyle{ดังนั้น\ \ x\ \in \ \big(0,\frac{1}{2}\big)\cup \big(1,2\big)}\)

b.)
\( \displaystyle{\begin{array}{lrcl}จาก\ \ &\sin (n\pi)&=&0&,n\in I\\จะได้&x-\sqrt{x^2-3x+2}&=&3n\\&x-3n&=&\sqrt{x^2-3x+2}\\&x^2-6nx+9n^2&=&x^2-3x+2\\&-3(2n+1)x&=&2-9n^2\\&x&=&\frac{2-9n^2}{-3(2n+1)} \end{array}} \)
ซึ่งพบว่า ไม่มีทางเป็นจำนวนเต็มครับ :D

nongtum 26 มิถุนายน 2005 16:43

ข้อยี่สิบถูกแล้วครับ การแก้สมการจำพวกนี้ไม่ระวังอาจผิดได้ง่ายๆนะเนี่ย

ในช่วงสัปดาห์สุดท้ายนี้ขอออก quickies อีกสักข้อละกัน

31. จำนวนใดมีค่ามากกว่า
a) 1714, 3111
b) \(\displaystyle\Large{2003^{2004^{2005}}, 2005^{2004^{2003}}}\)

Deadline สำหรับคำถามในส่วนที่เหลือ เอาเป็นวันที่ 30 มิถุนายน 2005 เวลา 23:59:59 ละกันครับ แต่หากใครต้องการจะถามทฤษฎีหรือไม่แน่ใจคำถามตรงไหนทั้งในและนอกกระทู้ ก็ถามได้เรื่อยๆครับ

gon 26 มิถุนายน 2005 17:07

มาถึงหน้าที่ 2 กันจนได้นะครับ. อยากให้น้อง ๆ แถวนี้ได้เป็นตัวแทนกันทุกคนเลย :) คว้าเหรียญทองเลขเหรียญต่อไปติดต่อกันทุกปี ใน My Maths ฉบับที่ 7 (ฉบับเดือนมะรืน) จะมีบทสัมภาษณ์ของ น้อง ธนสิน ด้วยนะครับ คงจะช่วยเติมไฟกันให้ได้ไม่มากก็น้อย ;) แต่ที่สำคัญที่สุดอยากใ้ห้ชอบเลขได้ตลอดไป :cool:

passer-by 27 มิถุนายน 2005 01:54

ที่คุณ nongtum เสนอมาก็ O.K. ครับ สรุปว่า ผมก็จะรอคำตอบจนหยดสุดท้ายถึงวันที่ 30 มิ.ย. แล้วพอเข้าวันที่ 1 ก.ค. คงได้เห็น คำอธิบายและคำตอบข้อที่ยัง blank อยู่แน่นอน

ส่วนตัวแล้ว ตอนนี้ที่รอเป็นพิเศษ คือน้องๆ ที่จะมาพิชิตข้อ 23 ,24
ถ้าทำได้และทำถูก ก็ถือว่า maths olympiad สสวท. หมูอู๊ดๆ แน่นอนครับ

R-Tummykung de Lamar 27 มิถุนายน 2005 23:49

ข้อ 23 ยากจังครับ
ถ้า x เป็นจำนวนจริงนี่ สามารถแสดงว่า x = 2 คำตอบเดียวได้ไม่ยากครับ แต่ ถ้าเป็นเชิงซ้อนนี่สิครับ ปวดหัวน่าดู

..จะพยายามคิดต่อไปครับ (หรือถ้าสิ้นคิดแล้วจะรอแนวคิดสวยๆครับผม :D )

passer-by 28 มิถุนายน 2005 03:23

สมการข้อ 23 มี 3 คำตอบครับ แถมยังเป็นจำนวนจริงล้วนๆ โดยมี x=2 เป็น solution ค่าหนึ่งในนี้
เห็นมั้ยครับว่า สมการที่ดูง่ายๆ แต่ทำจริงๆอาจจะอำมหิตมากๆ :D

warut 28 มิถุนายน 2005 06:35

คุณ passer-by หมายถึงในข้อ 23. ต้องแก้สมการหาว่า\[x=-\frac{1}{3}-
\frac{\sqrt7}{3}\left(\cos\left(\frac{\tan^{-1}(3\sqrt{3})}{3}\right)-
\sqrt3\sin\left(\frac{\tan^{-1}(3\sqrt{3})}{3}\right)\right)\]\[=-0.4450418679\dots\]ออกมาด้วยเหรอครับ :eek:

passer-by 28 มิถุนายน 2005 18:08

ถ้าทำแบบ manual คำตอบที่คุณ warut เขียน สามารถ simplify ให้สั้นกว่านี้ได้ครับ รู้สึกว่าตอนผม check จาก matlab ก็จะได้คำตอบยืดยาวมากๆ แบบนี้เหมือนกัน

อีก 2 คำตอบที่เหลือ ตอบยังไงก็ได้ครับ จะติด square root จะติด sin cos ตามสบายเลยครับ แต่อยากได้ที่กระชับที่สุด เพราะตอนนี้ 2 คำตอบที่ผมมีในมือ เขียนได้สั้นมากๆ ครับ :)

warut 28 มิถุนายน 2005 19:21

โอ้ว...จริงด้วยแฮะ ผมสะเพร่าอีกแล้ว ไม่ได้เช็คให้รอบคอบก่อน :p
ขอบคุณคุณ passer-by สำหรับคำชี้แนะครับ

passer-by 01 กรกฎาคม 2005 00:55

ได้เวลาเฉลยแล้ว

16. ยึด 1.5 = 3/2 ไว้

16.1 เพราะ\(\large (\frac{3}{2})^{2}> 2 \) ดังนั้น
\(\large \frac{3}{2}>\sqrt{2} \)

16.2 เพราะ\(\large (\frac{3}{2})^{3}> 3 \) ดังนั้น
\(\huge \frac{3}{2}>\sqrt[3]{3} \)

16.3 เพราะ\(\large 3^{2} > 2^{3}\) ดังนั้น เมื่อ take log ฐาน 2 จะได้
\(\large log_{2}3 > \frac{3}{2} \)
Similarly, เมื่อ take log ฐาน 3 ก็จะได้
\(\large \frac{2}{3}>log_{3}2 \) แต่ 3/2 > 2/3 ดังนั้น \(\large \frac{3}{2}>log_{3}2 \)

จาก 16.1-16.3 สรุปได้ว่า ค่ามากสุด คือ log23

23. ข้อนี้ ต้องวิเคราะห์ก่อน solve เนื่องจาก R.H.S. ทำให้เราทราบว่า x>-2

ถ้า x> 2 จะได้ \[ \huge \begin {array} {rcl} x^{3}-3x = \frac{x^{3}+3x(x-2)(x+2)}{4}\\ > \frac{x^{3}}{4} > \sqrt{x+2} \end{array}\]

สำหรับอสมการท้ายสุด อาจ พิสูจน์ได้ดังนี้
เพราะf(x)=x6-16x-32 เป็น increasing function บน (2,) โดยจะเห็นว่า
f(2)=0 สรุปว่า x6-16(x+2) >0 ซึ่ง equivalent กับ อสมการท้ายสุด

สรุปว่า ตอนนี้ คำตอบ อยู่ในช่วง [-2,2] Let x= 2cosq ( 0qp) Substitute in equation and after deriving step by step... Finally we obtain the following
2 cos3 q= 2cos(q/2)
ดังนั้น 3q ( q/2) = 2np for some integer n และจากการ แทนค่า พบว่า ที่ n=0,1 จะได้คำตอบของสมการคือ
x=2 (คำตอบนี้ของน้อง tummykung) , 2cos(4p/5) , 2cos (4p/7) (คำตอบนี้ของคุณ warut)

24. ข้อนี้ ไม่รู้ว่า ผมเขียนโจทย์เคลียร์หรือเปล่า ที่บอกว่าสัมประสิทธิ์ทุกตัวเป็น 1 ผมหมายความว่า พหุนามนี้ ต้องมี xn ,xn-1 ไล่ไปจนถึงเทอมที่เป็นค่าคงที่ โดย ส.ป.ส. เป็น 1

Let p(x) = xn +axn-1+bxn-2+...+c

Consider sum square of all real roots = a2 -2b

Apply AM-GM inequality to all square of roots, so

\(\huge \frac{a^{2}-2b}{n}\geq\sqrt[n]{c^{2}} \)
Since a,b,c = 1 , we get n3 and this completes the proof.


25. ขออธิบายสั้นๆ นะครับ
สังเกตว่า 13 หาร F6 , F13 ลงตัวและ โดย induction จะได้ 13 หาร F7k-1 ลงตัว เมื่อ k =1,2,3,... สรุปว่าข้อนี้ มี14 จำนวน

26. L.H.S. ของสมการ จัดรูปใหม่ เป็น \( \large cos50^{\circ} \) (square root ตัวหน้าและตัวหลัง คือ \(\large 2-sin^{2}25^{\circ} \) และ \(\large 2-cos^{2}25^{\circ} \) ตามลำดับ)
และเพราะ \( \large cos( \frac{\pi}{4}+\theta) =\frac{1}{\sqrt{2}}(cos\theta -sin\theta) \)
square both sides and let \( \large \theta= 5^{\circ} \) ,we obtain

\[ \large k=cos^{2}50^{\circ}=0.5(1-sin10^{\circ})=0.4132 \]

ท้ายที่สุด ขอให้น้องๆที่จะสอบโอลิมปิก เสาร์หรืออาทิตย์นี้ โชคดี ทุกคนครับ ;)

nongtum 01 กรกฎาคม 2005 03:28

แหะๆๆ เสียเวลาทำรูปนิดหน่อย แต่มาแล้วครับ

เฉลย(sketch)
17. จากรูปด้านล่าง เราจะแสดงว่ามุม BPC=(a+b)/2 (อันหมายถึงคำตอบเป็น 68 องศา)
เนื่องจาก GC CB BE เป็นเส้นสัมผัสวงกลม เราจะได้สี่เหลี่ยม PGCG และ PEBF cyclic
สามเหลี่ยม BEF และสามเหลี่ยม BED เป็นสามเหลี่ยมหน้าจั่วที่มีมุมที่ฐานเป็น b/2 และ a/2 ตามลำดับ
ดังนั้น \(B\hat{P}C=B\hat{E}F+F\hat{G}C=(a+b)/2\)

18. a) \(Sum=\frac{7}{6}(7^{2004}-1)\ \bigwedge\ 7^4\equiv1\pmod{100} \Rightarrow 100|((7^4)^{501}-1)\)
b) \(Sum=\frac{5}{4}(5^{150}-1)\ \bigwedge\ 5\equiv-1\pmod 3 \Rightarrow 6|(3^{150}-1)\) และโดย little Fermat จะได้ \(5^{30}\equiv 1\pmod {31}\) ดังนั้น \(31\times6=186|(5^{150}-1)\)

21. นิยาม: ให้ a,m,n เป็นจำนวนเต็มบวก #2(a)=k, n!=2km, (m,2)=1 ดังนั้น
\[#2(a)=\sum_{i=1}^{\infty}\lfloor\frac{a}{2^i}\rfloor\]
\(\Rightarrow\ #2(2048)=2^{10}+2^9+\cdots+1=2047\)
ขาด 2 ไปอีก 501\(<511=2^8+2^7+\cdots+1=#2(512)\) ตัว
ดังนั้น \(2047<n< 2047+512=2559\) ดังนั้น เราจะทดสอบเลขในช่วงนี้บางตัว ดังนี้
\(#2(2559)=#2(2558)=1279+639+\cdots+2+1=2549\)
\(#2(2557)=#2(2556)=1278+639+\cdots+2+1=2548\)
\(#2(2555)=#2(2554)=1277+638+\cdots+2+1=2546\)
ดังนั้น n=2556

27. (ดูรูปประกอบ) พื้นที่สามเหลี่ยมใหญ่ = \(1+\frac{1}{2}(x+\frac{1}{x})\ge2\)

28. a) ตามโจทย์จะไม่มีการเรียงใดที่เรียงแล้วหารด้วย 11 ลงตัว(นั่นคือ P=0) เพราะ 1+2+3+4+5+6=21 เป็นเลขคี่ (ลองนึกดูต่อนะครับว่าทำไม)
b) \(21=2\cdot9+3\) ดังนั้น P=1

29. เลข 1 หลัก มีแค่ 2 ตัวเดียวที่สอดคล้องเงื่อนไข (1+0)
เลข 2 หลัก มีแค่ 20 และ 11 ที่สอดคล้องเงื่อนไข (1+1)
เลข 3 หลัก มีแค่ 200 และ 101, 110 ที่สอดคล้องเงื่อนไข (1+2)
เลข 4 หลัก มีแค่ 2000 และ 1001, 1010, 1100 ที่สอดคล้องเงื่อนไข (1+3)
\(\vdots\)
เลข 2005 หลัก มีแค่ \(2\underbrace{0\ldots{}0}_{2004\ 0's}\) และ \(1\underbrace{0\ldots{}0}_{2003\ 0's}1,\ \ldots,11\underbrace{0\ldots{}0}_{2003\ 0's}\) ที่สอดคล้องเงื่อนไข (1+2004)
ดังนั้น มีจำนวนที่สอดคล้องเงื่อนไขทั้งหมด \(1+2+\ldots+2004+\underbrace{(1+\ldots+1)}_{2005\ 1's}=2011015\) ตัว

31. a) \(31^{11}<32^{11}=2^{55}<2^{56}=16^{14}<17^{14}\)
b) \(\displaystyle\large{\begin{array}{rl}
\log2003^{2004^{2005}}&=2005 \log2003^{2004}=2005\cdot2004\log2003\\
&\ >2004\cdot2003\log2005=2003 \log2005^{2004}=\log2005^{2004^{2003}}
\end{array}}\)

ขอติดวิธีทำข้อสิบไว้แป้บนึงครับ แล้วจะกลับมาเฉลยทีหลัง (คำตอบคือ 1333333332.3d, |d|<1)
หากไม่เข้าใจหรือสงสัยตรงไหน ถามได้ครับ
Viel Erfolg นะครับ ;)

EDIT4: แก้ข้อ 29 ตามคำท้วงของคุณ Passer-by (Thank you ^^')

passer-by 01 กรกฎาคม 2005 05:26

รู้สึกว่า ข้อ 29 ของ คุณ nongtum จะตกกรณีเลข 2005 หลัก ไป 1 กรณี
แล้วก็บรรทัดที่เป็นคำตอบ L.H.S. ไม่เท่ากับ R.H.S. นะครับ

สำหรับข้อนี้ ผมมี alternative solution อีกวิธีครับ

เพราะมี 2 กรณี ที่ผลบวกเลขโดดเป็น 2 คือ
(i) มี 2 ตัวเดียว (ที่เหลือ (ถ้ามี) เป็น 0 เท่านั้น)
พิจารณา จำนวน 2005 หลัก (ให้ 0 นำหน้าได้)
ต้องการ 2 ตัวเดียว ที่เหลือเป็น 0 จะได้วิธีสร้างจำนวนดังกล่าว \( \large \frac{2005!}{2004!1!}\) วิธี

(ii) มี 1 อยู่ 2จำนวน (ที่เหลือ(ถ้ามี) เป็น 0 เท่านั้น)
พิจารณา จำนวน 2005 หลัก (ให้ 0 นำหน้าได้)
ต้องการ 1 อยู่ 2จำนวน ที่เหลือเป็น 0 จะได้วิธีสร้างจำนวนดังกล่าว \( \large \frac{2005!}{2003!2!}\) วิธี

2 วิธี รวมกันได้ 20051003 จำนวน

nongtum 01 กรกฎาคม 2005 07:19

ขอสารภาพว่า
1. เบลอนิดหน่อยตอนคิดข้อ 29 ทั้งที่ก้ไม่ได้ยากอะไรเลย ขอบคุณคุณ Passer-by อีกครั้งสำหรับคำท้วงติงครับ(แก้แล้วจ้า)
2. ไม่น่าปล่อยข้อ 10 มาเลย เพราะยาวและยากเกินเหตุ ที่จะแสดงให้ดูต่อไปนี้เป็นแนวคิดคร่าวๆเท่านั้น
หากสนใจต้นฉบับ กรุณาไปหาอ่านได้จากหนังสือ Inequalities ของ P.P.Korovkin ของ Mir Publishers ครับ

ข้อสิบ (ย้ำอีกทีว่าเป็นแนวคิดคร่าวๆ)
โดยใช้ข้อเท็จจริงที่ว่า
\[
\sum_{i=1}^n \frac{1}{i^\alpha}
=\frac{2^\alpha}{2-2^\alpha}\sum_{i=1}^n \frac{1}{(n+i)^\alpha}
-\frac{2^\alpha}{2-2^\alpha}\sum_{j=1}^{2n} \frac{(-1)^{j-1}}{(j)^\alpha}
\] จะได้ว่าผลรวม S ที่ต้องการคือ
\[
\underbrace{\frac{2^{1/4}}{2-2^{1/4}}\sum_{i=1}^{10^{12}} \frac{1}{(10^{12}+i)^{1/4}}}_{=:A}
-\underbrace{\frac{2^{1/4}}{2-2^{1/4}}\sum_{j=1}^{2\cdot10^{12}} \frac{(-1)^{j-1}}{(j)^{1/4}}}_{=:B}
\]
เราจะใช้อสมการ
\[
\frac{(2n+1)^{1-\alpha}-(n+1)^{1-\alpha}}{1-\alpha}
< \sum_{i=1}^{n}\frac{1}{(n+i)^\alpha}
< \frac{(2n)^{1-\alpha}-n^{1-\alpha}}{1-\alpha}
\]
เมื่อ \(0<\alpha<1\) ประมาณค่าของผลรวม A ดังนี้
\[
\frac{2^{1/4}}{2-2^{1/4}}
\cdot\frac{(2\cdot10^{12})^{3/4}-(10^{12})^{3/4}}{1-\frac{1}{4}}
=\frac{4}{3}\cdot10^9
\]
และใช้ทฤษฎีบทต่อไปนี้ประมาณค่า B: ให้ \(x_1>x_2>\ldots>x_n\) จะได้ว่า
\[0<\sum_{i=1}^n (-1)^{i-1}x_i<x_1\]
จากทฤษฎีบทนี้เราจะได้ว่า B เป็นบวกและไม่มากกว่า A และเนื่องจากเทอมน้อยกว่าสอง จะได้อีกว่า
\[\frac{4}{3}\cdot10^9-2<S<\frac{4}{3}\cdot10^9\]
ทางซ้ายและขวาสุดของอสมการนี้ต่างกันอยู่ 2 และต่างจากผลรวมที่เราต้องการน้อยกว่า 2 ค่ากลาง \(\frac{4}{3}\cdot10^9-1\) ต่างจากผลรวมที่ต้องการไม่เกิน 1
นั่นคือ ผลรวมที่ต้องการมีค่าเป็น
\[\frac{4}{3}\cdot10^9-1\pm{}d=133333332.3\pm{}d,\qquad |d|<1 \]
ทั้งนี้ เราสังเกตว่าผลรวมที่ได้ค่อนข้างแม่นยำ เพราะ relative error < 10-7

phew... น้องคนไหนสอบเสร็จแล้ว อย่าลืมเอาข้อสอบมาปล่อยที่นี่ด้วยนะครับ :D

warut 01 กรกฎาคม 2005 10:06

ผมขอเสนอวิธีทำข้อ 10. ที่ผมคิดว่าง่ายกว่ามากโดยใช้หลักการอันเป็นที่มาของ integral test สำหรับตรวจสอบการลู่เข้าของอนุกรมดังนี้ครับ

ให้\[f(x)=\frac{1}{\sqrt[4]x}=
x^{-1/4}\]จะเห็นว่า f(x) เป็น strictly decreasing function เมื่อ x > 0 ดังนั้น\[f(m+1)\quad<\quad\int_m^{m+1}f(x)\,dx\quad<\quad
f(m)\]เราจึงได้ว่า\[\sum_{k=m+1}^{n}f(k)\quad<\quad
\int_m^nf(x)\,dx\quad<\quad
\sum_{k=m}^{n-1}f(k)\]โดยที่ m, n เป็นจำนวนเต็มบวกและ m < n (ผมไม่ได้แสดงละเอียดเพราะถ้าใครจำพิสูจน์ของ integral test ได้ก็คงจะเข้าใจได้ไม่ยากนัก แต่ถ้าไม่เคลียร์ตรงไหนก็ถามได้นะครับ)

ถ้า m = 1 และ n = 1012 เราจะได้ว่า\[\int_m^nf(x)\,dx=
\left[\frac{4}{3}x^{3/4}\right]_1^{10^{12}}=
1333333332\]จากอสมการทางซ้ายเราจึงได้ว่า\[\sum_{k=1}^{10^{12}}f(k)=1+\sum_{k=2}^{10^{12}}f(k)<1333333332+1=
1333333333\]และจากอสมการทางขวาเราจึงได้ว่า\[\sum_{k=1}^{10^{12}}f(k)=f(10^{12})+\sum_{k=1}^{10^{12}-1}f(k)>1333333332+0.001=
1333333332.001\]สรุปได้ว่า\[1333333332.001\quad<\quad\sum_{k=1}^{10^{12}}\frac{1}{\sqrt[4]k}\quad<\quad
1333333333\]จะเห็นว่ามี relative error ที่ดีขึ้นประมาณ 2 เท่าครับ

warut 03 กรกฎาคม 2005 05:13

อ้างอิง:

ข้อความเดิมของคุณ passer-by:
25. ขออธิบายสั้นๆ นะครับ
สังเกตว่า 13 หาร F6 , F13 ลงตัวและ โดย induction จะได้ 13 หาร F7k-1 ลงตัว เมื่อ k =1,2,3,... สรุปว่าข้อนี้ มี14 จำนวน

ถ้าคุณ passer-by มีโอกาสช่วยอธิบายการ induction แบบยาวๆให้ด้วยนะครับ (เพราะผมยังทำไม่ได้น่ะ :p)

ผมมีข้อสังเกตเกี่ยวกับโจทย์ข้อนี้คือ อย่างแรกนิยามของลำดับ Fibonacci แตกต่างจากที่ใช้กันโดยทั่วไป (F0 = 0, F1 = F2 = 1) ไม่แน่ใจว่าเป็นความจงใจรึเปล่า อีกอย่างคือผมคิดว่าการพิสูจน์ยังไม่สมบูรณ์นะครับ เพราะเราต้องพิสูจน์ด้วยว่า \(13\not|\,F_n\) เมื่อ n เป็นค่าอื่นๆที่ไม่ได้อยู่ในรูป 7k - 1


เวลาที่แสดงทั้งหมด เป็นเวลาที่ประเทศไทย (GMT +7) ขณะนี้เป็นเวลา 02:47

Powered by vBulletin® Copyright ©2000 - 2024, Jelsoft Enterprises Ltd.
Modified by Jetsada Karnpracha